Which best describes the function represented by the
table?
Х
-2
2
4
6
Y у
-5
5
10
15
O direct variation; k = 33 를
O direct variation; k = 5
- 를
O inverse variation; k = 10
direct variation; k = 1
10

Answers

Answer 1

Answer:

Direct variation

[tex]k = 2.5[/tex]

Step-by-step explanation:

Given

The attached table

Required

The type of variation

First, we check for direct variation using:

[tex]k = \frac{y}{x}[/tex]

Pick corresponding points on the table

[tex](x,y) = (-2,-5)[/tex]

So:

[tex]k = \frac{-5}{-2} = 2.5[/tex]

[tex](x,y) = (4,10)[/tex]

So:

[tex]k = \frac{10}{4} = 2.5[/tex]

[tex](x,y) = (6,15)[/tex]

So:

[tex]k = \frac{15}{6} = 2.5[/tex]

Hence, the table shows direct variation with [tex]k = 2.5[/tex]


Related Questions

Find a power series representation for the function. (Assume a>0. Give your power series representation centered at x=0 .)
f(x)=x2a7−x7

Answers

Answer:

Step-by-step explanation:

Given that:

[tex]f_x = \dfrac{x^2}{a^7-x^7}[/tex]

[tex]= \dfrac{x^2}{a^7(1-\dfrac{x^7}{a^7})}[/tex]

[tex]= \dfrac{x^2}{a^7}\Big(1-\dfrac{x^7}{a^7} \Big)^{-1}[/tex]

since  [tex]\Big((1-x)^{-1}= 1+x+x^2+x^3+...=\sum \limits ^{\infty}_{n=0}x^n\Big)[/tex]

Then, it implies that:

[tex]\implies \dfrac{x^2}{a^7} \sum \limits ^{\infty}_{n=0} \Big(\Big(\dfrac{x}{a} \Big)^{^7} \Big)^n[/tex]

[tex]= \dfrac{x^2}{a^7} \sum \limits ^{\infty}_{n=0} \Big(\dfrac{x}{a} \Big)^{^{7n}}[/tex]

[tex]= \dfrac{x^2}{a^7} \sum \limits ^{\infty}_{n=0} \Big(\dfrac{x^{7n}}{a^{7n}} \Big)}[/tex]

[tex]\mathbf{= \sum \limits ^{\infty}_{n=0} \dfrac{x^{7n+2}}{a^{7n+7}} }}[/tex]

find the measures of m and n.

Answers

Answer:

m = 4

n = 5

Step-by-step explanation:

[tex]m + 8 = 3m\\\\m - 3m = - 8\\\\-2m = - 8\\\\m = 4[/tex]

[tex]2n - 1 = 9 \\\\2n = 9 + 1\\\\2n = 10\\\\n = 5[/tex]

what is the range of the funcion y=x^2

Answers

Answer:

Range = [0, infinity)

Step-by-step explanation:

Minimum point of the graph is at (0,0) and it is a u shaped graph. Hence, range is 0 inclusive to infinity

The range of the function y = x2 is all real numbers, y, such that y ≥ 0. In the case of the function y = x2, the domain consists of all real

Find the value of x.

Answers

Answer:

x = 3

Step-by-step explanation:

A midsegment in a trapezoid is formed when one connects the midpoints of the two legs (non-parallel sides) in a trapezoid. The midsegment theorem states that the length of the midsegment is equal to the average of the two bases (that is the parallel sides).

One can apply the midsegment theorem here by stating the following;

[tex]\frac{(YZ)+(TM)}{2}=PW[/tex]

Substitute,

[tex]\frac{23+11x+2}{2}=29[/tex]

Simplify,

[tex]\frac{25+11x}{2}=29[/tex]

Inverse operations,

[tex]\frac{25+11x}{2}=29[/tex]

[tex]25+11x=58\\\\11x = 33\\\\x = 3[/tex]

4,3,5,9,12,17,...what is the next number?

Answers

Answer:

The next number is going to be 21

Answer:

19

Step-by-step explanation:

4 even number

3,5,7 odd numbers

14 even

17, 19, 21 even

find the exact value cos5pi/6

Answers

Answer:

[tex] - \frac{ \sqrt{3} }{2} [/tex]

Step-by-step explanation:

Unit circle

SCALCET8 3.9.015. A street light is mounted at the top of a 15-ft-tall pole. A man 6 ft tall walks away from the pole with a speed of 4 ft/s along a straight path. How fast is the tip of his shadow moving when he is 35 ft from the pole

Answers

Answer:

[tex]X=6.67ft/s[/tex]

Step-by-step explanation:

From the question we are told that:

Height of pole [tex]H_p=15[/tex]

Height  of man [tex]h_m=6ft[/tex]

Speed of Man [tex]\triangle a =4ft/s[/tex]

Distance from pole [tex]d=35ft[/tex]

Let

Distance from pole to man=a

Distance from man to shadow =b

Therefore

 [tex]\frac{a+b}{15}=\frac{b}{6}[/tex]

 [tex]6a+6b=15y[/tex]

 [tex]2a=3b[/tex]

Generally the equation for change in velocity is mathematically given by

 [tex]2(\triangle a)=3(\triangle b )[/tex]

 [tex]2*4=3(\triangle b)[/tex]

 [tex]\triangle a=\frac{8}{3}[/tex]

Since

The speed of the shadow is given as

 [tex]X=\triangle b+\triangle a[/tex]

 [tex]X=4+8/3[/tex]

 [tex]X=6.67ft/s[/tex]

Find the direction cosines and direction angles of the vector. (Give the direction angles correct to the nearest degree.) c, c, c , where c > 0

Answers

Answer:

cos(∝) = 1/√3

cos(β) = 1/√3

cos(γ) = 1/√3

∝ = 55°

β = 55°

γ = 55°

Step-by-step explanation:

Given the data in the question;

vector is z = < c,c,c >

the direction cosines and direction angles of the vector = ?

Cosines are the angle made with the respect to the axes.

cos(∝) = z < 1,0,0 > / |z|

so

cos(∝) = < c,c,c > < 1,0,0 > / √[c² + c² + c²] = ( c + 0 + 0 ) / √[ 3c² ]

cos(∝) = c / √[ 3c² ] = c / c√3 = 1/√3

∝ = cos⁻¹( 1/√3 ) = 54.7356° ≈ 55°

cos(β) = < c,c,c > < 0,1,0 > / √[c² + c² + c²] = ( 0 + c + 0 ) / √[ 3c² ]

cos(β) = c / √[ 3c² ] = c / c√3 = 1/√3

β = cos⁻¹( 1/√3 ) = 54.7356° ≈ 55°

cos(γ) = < c,c,c > < 0,0,1 > / √[c² + c² + c²] = ( 0 + 0 + c ) / √[ 3c² ]

cos(γ) = c / √[ 3c² ] = c / c√3 = 1/√3

γ = cos⁻¹( 1/√3 ) = 54.7356° ≈ 55°

Therefore;

cos(∝) = 1/√3

cos(β) = 1/√3

cos(γ) = 1/√3

∝ = 55°

β = 55°

γ = 55°

PLSHELPASAPDFFFFFFFFFFFFFFFFFFFFFFFFFF

Answers

im struggling with the same one

At the Fidelity Credit Union, a mean of 5.8 customers arrive hourly at the drive-through window. What is the probability that, in any hour, more than 5 customers will arrive

Answers

Answer:

0.5217

Step-by-step explanation:

P(more than 5 customer arrive):

P(X>=6)=1-P(X<=5)=  1-∑x=0x e-λ*λx/x!= 0.5217

A town recently dismissed 5 employees in order to meet their new budget reductions. The town had 4 employees over 50 years of age and 16 under 50. If the dismissed employees were selected at random, what is the probability that no more than 1 employee was over 50

Answers

Answer:

0.7513 = 75.13% probability that no more than 1 employee was over 50

Step-by-step explanation:

The employees are chosen from the sample without replacement, which means that the hypergeometric distribution is used to solve this question.

Hypergeometric distribution:

The probability of x successes is given by the following formula:

[tex]P(X = x) = h(x,N,n,k) = \frac{C_{k,x}*C_{N-k,n-x}}{C_{N,n}}[/tex]

In which:

x is the number of successes.

N is the size of the population.

n is the size of the sample.

k is the total number of desired outcomes.

Combinations formula:

[tex]C_{n,x}[/tex] is the number of different combinations of x objects from a set of n elements, given by the following formula.

[tex]C_{n,x} = \frac{n!}{x!(n-x)!}[/tex]

In this question:

4 + 16 = 20 employees, which means that [tex]N = 20[/tex]

4 over 50, which means that [tex]k = 4[/tex]

5 were dismissed, which means that [tex]n = 5[/tex]

What is the probability that no more than 1 employee was over 50?

Probability of at most one over 50, which is:

[tex]P(X \leq 1) = P(X = 0) + P(X = 1)[/tex]

In which

[tex]P(X = x) = h(x,N,n,k) = \frac{C_{k,x}*C_{N-k,n-x}}{C_{N,n}}[/tex]

[tex]P(X = 0) = h(0,20,5,4) = \frac{C_{4,0}*C_{16,5}}{C_{20,5}} = 0.2817[/tex]

[tex]P(X = 1) = h(1,20,5,4) = \frac{C_{4,1}*C_{16,4}}{C_{20,5}} = 0.4696[/tex]

Then

[tex]P(X \leq 1) = P(X = 0) + P(X = 1) = 0.2817 + 0.4696 = 0.7513[/tex]

0.7513 = 75.13% probability that no more than 1 employee was over 50

Describe the motion of a particle with position (x, y) as t varies in the given interval. (For each answer, enter an ordered pair of the form x, y.) x = 1 + sin(t), y = 3 + 2 cos(t), π/2 ≤ t ≤ 2π

Answers

Answer:

The motion of the particle describes an ellipse.

Step-by-step explanation:

The characteristics of the motion of the particle is derived by eliminating [tex]t[/tex] in the parametric expressions. Since both expressions are based on trigonometric functions, we proceed to use the following trigonometric identity:

[tex]\cos^{2} t + \sin^{2} t = 1[/tex] (1)

Where:

[tex]\cos t = \frac{y-3}{2}[/tex] (2)

[tex]\sin t = x - 1[/tex] (3)

By (2) and (3) in (1):

[tex]\left(\frac{y-3}{2} \right)^{2} + (x-1)^{2} = 1[/tex]

[tex]\frac{(x-1)^{2}}{1}+\frac{(y-3)^{2}}{4} = 1[/tex] (4)

The motion of the particle describes an ellipse.

The solution of this equation has an error. Which of the following steps has the error? 18 − (3x + 5) = 8
Step 1: 18 − 3x + 5 = 8
Step 2: -3x + 23 = 8
Step 3: -3x = -15
Step 4: x = 5



Step 1 Step 2 Step 3 Step 4. ?

Answers

Answer:

Step 1

Because the number in front of the bracket is 1 and it is also affected by the negative sign(-),5 is supposed to be negative not positive because (negative by positive is negative)

And since the first step has an error in it,the remaining steps would also be wrong.

Suppose f(x,y,z) = x2 + y2 + z2 and W is the solid cylinder with height 7 and base radius 2 that is centered about the z-axis with its base at z = −2. Enter θ as theta.

A) As an iterated integral, ∭WfdV = ∫BA∫DC∫FE dzdrdθ with limits of integration.

B) Evaluate the integral.

Answers

In cylindrical coordinates, W is the set of points

W = {(r, θ, z) : 0 ≤ r ≤ 2 and 0 ≤ θ ≤ 2π and -2 ≤ z ≤ 5}

(A) Then the integral of f(x, y, z) over W is

[tex]\displaystyle\iiint_W(x^2+y^2+z^2)\,\mathrm dV = \int_0^{2\pi}\int_0^2\int_{-2}^5r(r^2+z^2)\,\mathrm dz\,\mathrm dr\,\mathrm d\theta[/tex]

(B)

[tex]\displaystyle \int_0^{2\pi}\int_0^2\int_{-2}^5r(r^2+z^2)\,\mathrm dz\,\mathrm dr\,\mathrm d\theta = 2\pi \int_0^2\int_{-2}^5(r^3+rz^2)\,\mathrm dz\,\mathrm dr \\\\\\= 2\pi \int_0^2\left(zr^3+\frac13rz^3\right)\bigg|_{z=-2}^{z=5}\,\mathrm dr \\\\\\= 2\pi \int_0^2\left(\frac{133}3r+7r^3\right)\,\mathrm dr \\\\\\= 2\pi \left(\frac{133}6r^2+\frac74r^4\right)\bigg|_{r=0}^{r=2} \\\\\\= 2\pi \left(\frac{110}3\right) = \boxed{\frac{220\pi}3}[/tex]

In the diagram, WZ=StartRoot 26 EndRoot.

On a coordinate plane, parallelogram W X Y Z is shown. Point W is at (negative 2, 4), point X is at (2, 4), point Y is at (1, negative 1), and point Z is at (negative 3, negative 1).

What is the perimeter of parallelogram WXYZ?

units
units
units
units

Answers

Answer:

[tex]P = 8 + 2\sqrt{26}[/tex]

Step-by-step explanation:

Given

[tex]W = (-2, 4)[/tex]

[tex]X = (2, 4)[/tex]

[tex]Y = (1, -1)[/tex]

[tex]Z = (-3,-1)[/tex]

Required

The perimeter

First, calculate the distance between each point using:

[tex]d = \sqrt{(x_1 - x_2)^2 + (y_1 -y_2)^2[/tex]

So, we have:

[tex]WX = \sqrt{(-2- 2)^2 + (4-4)^2 } =4[/tex]

[tex]XY = \sqrt{(2- 1)^2 + (4--1)^2 } =\sqrt{26}[/tex]

[tex]YZ = \sqrt{(1- -3)^2 + (-1--1)^2 } =4[/tex]

[tex]ZW = \sqrt{(-3--2)^2 + (-1-4)^2 } =\sqrt{26}[/tex]

So, the perimeter (P) is:

[tex]P = 4 + \sqrt{26} + 4 + \sqrt{26}[/tex]

[tex]P = 8 + 2\sqrt{26}[/tex]

Answer:

its D.

Step-by-step explanation:

took test

The original price of a set lunch was 30 dollars. It is now sold at a 20%
discount. There is an extra discount of 10% for students. How much
should a student pay to order a set lunch?

Answers

Find out the total discount percentage that students receive.

20% + 10% = 30%

30/100 x 30 = 9 dollars

30 - 9 = 21 dollars (Final answer)
20% of $30 = $6
30 - 6 = $24
10% of $24 = $2.40
24 - 2.40 = $21.60 cost of student lunch

You have to do each percent separate to get the cost.
If you just take 30% off from the original cost of $30 it will be the wrong answer.

What is the solution set of the equation x2+3*-4=6

Answers

Answer:

x=9

Step-by-step explanation:

Roulette is a casino game that involves spinning a ball on a wheel that is marked with numbered squares that are red, black, or green. Half of the numbers 1 - 36 are colored red and half are black and the numbers 0 and 00 are green. Each number occurs only once on the wheel. What is the probability of landing on an even number and a number greater than 17? (A number is even if it is divisible by 2. 0 and 00 are considered even as well.)​

Answers

Answer:

the wording (punctuation) of the question can lead to different interpretations....

I assume that the question was >17 & even which is "5/19",

BUT... it can also be read as two questions

first >17 which is "10/19"

and second an even number which is "9/19"

BUT !!! I think that the question answer is 5/19

Step-by-step explanation:

Even Number  = 18/38 = 9/19

greater 17 = 20/38 = 10/19

Even & greater 17 = 10/38 = 5/19

9. Mariah has 28 centimeters of reed
and 10 meters of reed for weaving
baskets. How many meters of reed
does she have? Write your answer as a
decimal and explain your answer.

Answers

10.28 , 28 cm in meters is 0.28. you have 10 meters therefore 10+.28= 10.28

A large container holds 4 gallons of chocolate milk that has to be poured into bottles. Each bottle holds 2 pints.
If the ratio of gallons to pints is 1: 8,
bottles are required to hold the 4 gallons of milk.

Answers

Answer:

64 Bottles

Step-by-step explanation:

that is the procedure above

please help i guess on a

Answers

Answer:

A = (2, 2)

B = (3, -1)

C = (-1, 0)

Step-by-step explanation:

To translate a point, you have to translate each individual point. At the bottom it shows <-2,3>, therefore you have to translate x 2 units to the left (because it's negative meaning the number is going away from 0, and 3 units to the right because 3 is a positive number.)

First Point A:

x: 4 - 2 = 2; y: -1 + 3 = 2

Second Point B:

x: 5 - 2 = 3; y: -4 + 3 = -1

Lastly, Point C:

x: 1 - 2 = -1, y: -3 + 3 = 0

I hope this helps!

What angles can you construct using just a pair of compasses and a ruler?

Answers

Answer:

By using a pair of compasses and a ruler you can draw all angles

Please help me out with these questions :

Answers

Answer:

Step-by-step explanation:

1. 3/7 x = 12

  3x = 84

    x = 28

2. 3x+ 6 = 39

   3x = 33

    x = 11

3. 1/3 x - 3/4 x = 15

    9x  - 4x  = 180

       x =  36

4.   1/4 x = x -21

      3/4 x = 21

      3x = 84

        x=28

5.    86-36 = 50

      50/2

       25

The asymptote of the function f(x) = 3x + 1 – 2 is . Its y-intercept is

Answers

Answer:

-1

Step-by-step explanation:

1-2=-1

y=mx+b

b= y intercept

Answer:

-1

Step-by-step explanation:

Find the solution of the differential equation that satisfies the given initial condition. (dP)/(dt)

Answers

Answer:

[tex]P = (\frac{1}{3}t^\frac{3}{2} + \sqrt 2 - \frac{1}{3})^2[/tex]

Step-by-step explanation:

Given

[tex]\frac{dP}{dt} = \sqrt{Pt[/tex]

[tex]P(1) = 2[/tex]

Required

The solution

We have:

[tex]\frac{dP}{dt} = \sqrt{Pt[/tex]

[tex]\frac{dP}{dt} = (Pt)^\frac{1}{2}[/tex]

Split

[tex]\frac{dP}{dt} = P^\frac{1}{2} * t^\frac{1}{2}[/tex]

Divide both sides by [tex]P^\frac{1}{2}[/tex]

[tex]\frac{dP}{ P^\frac{1}{2}*dt} = t^\frac{1}{2}[/tex]

Multiply both sides by dt

[tex]\frac{dP}{ P^\frac{1}{2}} = t^\frac{1}{2} \cdot dt[/tex]

Integrate

[tex]\int \frac{dP}{ P^\frac{1}{2}} = \int t^\frac{1}{2} \cdot dt[/tex]

Rewrite as:

[tex]\int dP \cdot P^\frac{-1}{2} = \int t^\frac{1}{2} \cdot dt[/tex]

Integrate the left hand side

[tex]\frac{P^{\frac{-1}{2}+1}}{\frac{-1}{2}+1} = \int t^\frac{1}{2} \cdot dt[/tex]

[tex]\frac{P^{\frac{-1}{2}+1}}{\frac{1}{2}} = \int t^\frac{1}{2} \cdot dt[/tex]

[tex]2P^{\frac{1}{2}} = \int t^\frac{1}{2} \cdot dt[/tex]

Integrate the right hand side

[tex]2P^{\frac{1}{2}} = \frac{t^{\frac{1}{2} +1 }}{\frac{1}{2} +1 } + c[/tex]

[tex]2P^{\frac{1}{2}} = \frac{t^{\frac{3}{2}}}{\frac{3}{2} } + c[/tex]

[tex]2P^{\frac{1}{2}} = \frac{2}{3}t^\frac{3}{2} + c[/tex] ---- (1)

To solve for c, we first make c the subject

[tex]c = 2P^{\frac{1}{2}} - \frac{2}{3}t^\frac{3}{2}[/tex]

[tex]P(1) = 2[/tex] means

[tex]t = 1; P =2[/tex]

So:

[tex]c = 2*2^{\frac{1}{2}} - \frac{2}{3}*1^\frac{3}{2}[/tex]

[tex]c = 2*2^{\frac{1}{2}} - \frac{2}{3}*1[/tex]

[tex]c = 2\sqrt 2 - \frac{2}{3}[/tex]

So, we have:

[tex]2P^{\frac{1}{2}} = \frac{2}{3}t^\frac{3}{2} + c[/tex]

[tex]2P^{\frac{1}{2}} = \frac{2}{3}t^\frac{3}{2} + 2\sqrt 2 - \frac{2}{3}[/tex]

Divide through by 2

[tex]P^{\frac{1}{2}} = \frac{1}{3}t^\frac{3}{2} + \sqrt 2 - \frac{1}{3}[/tex]

Square both sides

[tex]P = (\frac{1}{3}t^\frac{3}{2} + \sqrt 2 - \frac{1}{3})^2[/tex]

Please help me >_< will give out brainliest

Answers

Answer:  1080

====================================================

Explanation:

We have an octagon because there are n = 8 sides. The diagram below shows one way to number the sides so you can count them efficiently (without missing any or double counting any).

----------------

Plug n = 8 into the formula below

S = 180(n-2)

S = 180(8-2)

S = 180(6)

S = 1080

The 8 interior angles add up to 1080 degrees.

Exactly how many planes contain points J, K, and N?
a - 0
b - 1
c - 2
d - 3

Answers

A- 0 ..... one plane contains k and n but there isn’t a plane that contains all three points

PLEASE HELP!!! Which number is a solution of the inequality x less-than negative 4? Use the number line to help answer the question. A number line going from negative 9 to positive 1.

Answers

Answer:

is it going to be 10.5

Step-by-step explanation:

I do not have any explanation

Answer: 0 (zero)

Step-by-step explanation:

Start Learning & start growing! edge2023

*DROPS THE MIC*

Find the perimeter of a football field which measures 90m by 60m

Answers

Hello!

[tex]\large\boxed{P = 300m}[/tex]

Use the following formula for the perimeter:

P = 2l + 2w, where:

l = length

w = width

Therefore:

P = 2(90) + 2(60)

Simplify:

P = 180 + 120 = 300 m

Answer:

well how about you use common sense 100 yards long on each side 200 yards then add 5o yards since the the that is how wide it is then add another 50 and you get 300 yards then convert that to meters

Sofia bought a clothes iron that was discounted 15% off of the original price of $35. What was the sale price of the clothes iron?

Answers

Answer:

35 - 0.15 * 35 so it is $29.75

Step-by-step explanation:

I got u

Answer:

$29.75

Step-by-step explanation:

15% = .15

.15 x 35 = 5.25

35 - 5.25 = 29.75

Other Questions
14.With earned value management (EVM), the Government can determine if a program is currently experiencing a/n Calculate the average atomic mass for element X $975, and there is 7 percent sales tax on the purchase.How much is the sales tax? number of ways you can wear 10 outfits to school each day in a 5 day week What do you think Joses grandmother meant by saying that the bread was too dense? What other things can you think of that are dense? Write on ONE of the following topics.1. A letter to a hotel manager complaining about their bad services. When a shift in ________ occurs, rational expectations hold that its impact on output and employment will only be minimal. Janna is using a cone-shaped cup to fill a cylindrical container. The cup has the same height and radius as the container. How many rimes will she have to fill the cone-shaped cup to completely fill the cylindrical container. A source emits sound at a fixed constant frequency f. If you run towards the source, the frequency you hear is A and B are two similar solids... Do government statisticians calculate GDP by simply adding up the total sales of all business firms in one year? Explain. Si se analizan muestras de Al2O3 en diversos laboratorios se encuentra que todas tienen 52,94% de aluminio (Al) y 47,06% de oxgeno (O). Este dato experimental corresponde a lo expresado por la ley de:A) La ley de las proporciones definidasB) La ley de las proporciones mltiples C) La ley de la conservacin de la masaD) La ley de la conservacin de la energa Help please. I'm stuck Calculate the moles of H3PO4 that reacted (8). Consult the coefficients in the balance chemical reaction to obtain the mole ratio. Show your calculation here. Xinhong Company is considering replacing one of its manufacturing machines. The machine has a book value of $45,000 and a remaining useful life of 5 years, at which time its salvage value will be zero. It has a current market value of $55,000. Variable manufacturing costs are $33,400 per year for this machine. Information on two alternative replacement machines follows. Alternative A Alternative BCost $ 119,000 $ 112,000 Variable manufacturing costs per year 23,000 10,200 Calculate the total change in net income if Alternative A, B is adopted. Should Xinhong keep or replace its manufacturing machine? If the machine should be replaced, which alternative new machine should Xinhong purchase? 2m^2-5m-3=0 by factorization Compute ????????, where ????=????2????+5????, ????=2????+????+3????. (Write your solution using the standard basis vectors ????, ????, and ????. Use symbolic notation and fractions where needed.) The number of chocolate chips in a bag of chocolate chip cookies is approximately normally distributed with mean of 1262 and a standard deviation of 118. Determine the 26th percentile for the number of chocolate chips in a bag. (b) Determine the number of chocolate chips in a bag that make up the middle 95% of bags. (c) What is the interquartile range of the number of chocolate chips in a bag of chocolate chip cookies? Plot point C in GeoGebra to verify that it indeed lies on AB. You can enter the coordinates in the Input window to plot the point . Also , verify the distances you calculated in question 3 using GeoGebra . Take a screenshot showing the distances displayed by GeoGebra , and paste it below . What is the value of k?